Main Page/Suggestions

If you want to edit something on the main page, just edit this page and the administrators will take your suggestions into consideration.


During AMC testing, the AoPS Wiki is in read-only mode. No edits can be made.

Problem of the Week

2008 AMC 10B, Problem 14

Triangle $OAB$ has $O=(0,0)$, $B=(5,0)$, and $A$ in the first quadrant. In addition, $\angle ABO=90^\circ$ and $\angle AOB=30^\circ$. Suppose that $OA$ is rotated $90^\circ$ counterclockwise about $O$. What are the coordinates of the image of $A$?

New to the AoPS Wiki?

If you're new to editing a wiki, we recommend starting with the tutorial. You can also check out what AoPS Wiki is not to better understand its scope.

Before you start editing, please familiarize yourself with the articles related to AoPS Wiki, especially the policies and guidelines. Additionally, take a moment to read why AoPS Wiki stands apart from other online resources.

AoPS Wiki is powered by MediaWiki software.